in the adjoining figure prove that<A+<B+<C+<D+<E is equal 180​

In The Adjoining Figure Prove That&lt;A+&lt;B+&lt;C+&lt;D+&lt;E Is Equal 180

Answers

Answer 1

It has been proved that the 5 star vertices have their sum of angles as A + B + C + D + E = 180°

How to find the sum of angles of a polygon?

The adjoining star contains a regular pentagon. Thus;

Sum of interior angles of the pentagon = (5 - 2) * 180 = 540°

Thus;

Each interior angle of the pentagon = 540/5 = 108°

Thus, each exterior angle = 180 - 108 = 72°

Then measure of the angle at the vertex = 180 - 72 - 72 = 36°

Thus, each angle at the vertices of the star have an angle of 36°.

There are 5 star vertices and so;

Sum of angles of 5 star vertices = 5 * 36 = 180°

Read more about Angles in a Polygon at; https://brainly.com/question/1592456

#SPJ1


Related Questions

solution.
5(4x + 7)-2x = ________+ 21

Answers

Answer:

18x + 14

Step-by-step explanation:

First, we need to distribute the 5:

 5(4x + 7) - 2x = __ + 21

 20x + 35 - 2x = __ + 21

Now, we can combine like terms:

 20x + 35 - 2x  = ___ + 21

  18x + 35 = ___ + 21

> subtract 21 from both sides to isolate "___"

 18x + 35 = __ + 21

          -21             -21

18x + 14 = ___

now that we have a solution, we should check that it works:

5(4x + 7) - 2x = ____ + 21

5(4x + 7) - 2x = 18x + 14 + 21  

{simplify both sides:}

20x + 35 -2x = 18x + 35

18x + 35 = 18x + 35

(TRUE)


So, we know our solution is 18x + 14

hope this helps!!

(8.6x107)-(9.1x10-8)simplify

Answers

Answer:

  85,999,999.999 999 909

Step-by-step explanation:

The expression represents the difference of a relatively large number and one that is relatively small. That difference is approximately the value of the large number. The exact value requires 17 digits for its proper expression. Most calculators and spreadsheets cannot display this many digits.

Standard form

The numbers in standard form are ...

  86,000,000 = 8.6×10^7

  0.000000091 = 9.1×10^-8

Difference

Their difference is ...

  86,000,000 -0.000000091 = 85,999,999.999 999 909

In scientific notation, this is ...

  8.599 999 999 999 990 9×10^7

A basketball coach recorded how many shots each of the 30 players at tryouts made in a set of 6 attempts.

Answers

Answer:

Step-by-step explanation:

what is the question?

Answer:

Median = A

Mean = B

Step-by-step explanation:

Khan Academy

f(x)=(2x−3)(x+6)(5x+6)f, left parenthesis, x, right parenthesis, equals, left parenthesis, 2, x, minus, 3, right parenthesis, left parenthesis, x, plus, 6, right parenthesis, left parenthesis, 5, x, plus, 6, right parenthesis has zeros at x=-6x=−6x, equals, minus, 6, x=-\dfrac{6}{5}x=− 5 6 ​ x, equals, minus, start fraction, 6, divided by, 5, end fraction, and x=\dfrac{3}{2}x= 2 3 ​ x, equals, start fraction, 3, divided by, 2, end fraction. What is the sign of fff on the interval -6

Answers

Answer:

566

Step-by-step explanation:

I got it right..................

URGENT PLEASE DUE TONIGHT ;(( functions

Answers

The value of (f/g)(-1) is 0, value of (g . f) (2) is  -3√3, the value of (g - f)(-1)  is √15, and the value of (g + f) (2) is √3 - 3

What is a function?

It is defined as a special type of relationship, and they have a predefined domain and range according to the function every value in the domain is related to exactly one value in the range.

We have two functions:

f(x) = 1 - x²

g(x) = √(11 - 4x)

f(-1) = 0

g(-1) = √15

f(2) = -3

g(2) = √3

(f/g)(-1) = f(-1)/g(-1) = 0/√15  = 0

(g . f) (2) = g(2)f(2) = (√3)(-3) = -3√3

(g - f)(-1) = g(-1) - f(-1) = √15 - 0 = √15

(g + f) (2) = g(2) + f(2) = √3 + (-3) = √3 - 3

Thus, the value of (f/g)(-1) is 0, value of (g . f) (2) is  -3√3, the value of (g - f)(-1)  is √15, and the value of (g + f) (2) is √3 - 3

Learn more about the function here:

brainly.com/question/5245372

#SPJ1

4. Write the function of a linear equation that includes the
points (1,3) and (2,9).

Answers

Answer:

y = 6x - 3

Step-by-step explanation:

the equation of a line in slope- intercept form ( linear function ) is

y = mx + c ( m is the slope and c the y- intercept )

calculate m using the slope formula

m = [tex]\frac{y_{2}-y_{1} }{x_{2}-x_{1} }[/tex]

with (x₁, y₁ ) = (1, 3 ) and (x₂, y₂ ) = (2, 9 )

m = [tex]\frac{9-3}{2-1}[/tex] = [tex]\frac{6}{1}[/tex] = 6 , then

y = 6x + c ← is the partial equation

to find c substitute either of the 2 points into the partial equation

using (1, 3 ) , then

3 = 6 + c ⇒ c = 3 - 6 = - 3

y = 6x - 3 ← equation of linear function


The sin (0) = - and lies in Quadrant III. Find the exact values of the sine
6'
and cosine of 20.

Answers

Answer:

Can you give exanation

I don't understand

A sequence is defined by the recursive function f(n + 1) = one-half(n). If f(3) = 9 , what is f(1) ?

Answers

The value of f(1) is 81

What is Recursive function?

Recursive Function is a function that repeats or uses its own previous term to calculate subsequent terms and thus forms a sequence of terms

Given:

f(n + 1) = 1/3* f(n)

f(3) = 9

let n=0

f(0+1) = 1/3 * f(0)

f(1) = 1/3 * f(0)

Let n=1

f(1+1) = 1/3 * 1/3*f(0)

f(2) = 1/9 * f(0)

let n=2

f(2+1) = 1/3 * 1/9* f(0)

f(3) = 1/27 * f(0)

Also, f(3) =9

So,

9 = 1/27 * f(0)

f(0) = 27 * 9

f(0) = 243

Now,

f(1) = 1/3 * f(0)

f(1) = 1/3/ 243

f(1) = 81

Learn more about this concept here:

https://brainly.com/question/14216181

#SPJ1

Lillian went shopping for a new phone because of a sale. The price on the tag was $20, but Lillian paid $16 before tax. Find the percent discount.pls help

Answers

Answer:

20 % discount

Step-by-step explanation:

Lilian got 4 dollars off of the original price    (20 - 16 = 4 )

4 / 20  * 100% = 20% off

The percentage of discount that Lillian gets because of sale is 20%.

What is percentage?

Percentage is a measurement to find value of given number out of hundred.

Given that,

The price of phone on tag = $20.

Lillian paid for phone = $16.

To find the percent discount on phone,

First find amount of discount = 20 - 16 = $4.

The percentage of discount = (4 / 20) x 100 = 20 %

Lillian gets 20% discount on new mobile.

To learn more on Percentage:

https://brainly.com/question/24120406

#SPJ2

A company making tires for bikes is concerned about the exact width of its cyclocross tires. The company has a lower specification limit of 22.8 millimeters and an upper specification limit of 23.1 millimeters. The standard deviation is 0.19 millimeters and the mean is 22.9 millimeters. What is the process capability index for the process? Note: Round your answer to 4 decimal places.

Answers

The process capability index for the process is 0.1754.

How to calculate the index?

The first sided specification limit will be:

= (Upper specification limit - mean)/(3 × standard deviation)

= (23.1 - 22.9)/(3 × 0.19)

= 0.2/0.57

= 0.3508

The second sided specification limit will be:

= (22.9 - 22.8)/(3 × 0.19)

= 0.1/0.57

= 0.1754

The process capability index for the process is 0.1754 wine it's the lower value.

Learn more about capability index on:

brainly.com/question/15734839

#SPJ12

What is the equation of the line that passes through the point (5,-2) and has a
slope of -2/5

Answers

-------------------------------------------------------------------------------------------------------------

Answer:  [tex]\textsf{y = -2/5x}[/tex]

-------------------------------------------------------------------------------------------------------------

Given:  [tex]\textsf{Passes through (5, -2) and slope of -2/5}[/tex]

Find:  [tex]\textsf{The equation that follows the details provided}[/tex]

Solution: We first need to plug into the point-slope form and after simplifying, distributing, and solving for y we will complete our equation.

Plug in the values

[tex]\textsf{y - y}_1\textsf{ = m(x - x}_1\textsf{)}[/tex][tex]\textsf{y - (-2) = -2/5(x - 5)}[/tex]

Distribute and simplify

[tex]\textsf{y + 2 = -2/5(x - 5)}[/tex][tex]\textsf{y + 2 = (-2/5 * x) + (-2/5 * (-5))}[/tex][tex]\textsf{y + 2 = -2/5x + 2}[/tex]

Subtract 2 from both sides

[tex]\textsf{y + 2 - 2 = -2/5x + 2 - 2}[/tex][tex]\textsf{y = -2/5x + 2 - 2}[/tex][tex]\textsf{y = -2/5x}[/tex]

Therefore, the final equation that follows the information that was provided is y = -2/5x.

Please help me out…

Polygon ____ and polygon ____ are similar to polygon 1.

Answers

Polygon 3 and 4 are similar to polygon 1.

Polygons will be called similar if they are exactly in the same shape, but can be in different sizes.

Here given, Some of the images are images of the polygon 1 from similarity transformations.

So we have to find the similar polygon from the given picture.

From picture it is clear that Polygon 2 is similar to polygon 1 as the size of polygon 2 has been decreased from polygon 1 but the shape of polygon 2 is the same as polygon 1 as we can their corresponding angles are also equal.

Also, polygon 3 is similar to polygon 1 as the size of polygon 3 has been decreased from polygon 1 but the shape of polygon 2 is the same as polygon 1 and reversed of polygon 1. So polygon 3 is of the same orientation as polygon 1 as we can their corresponding angles are also equal.

So from above, it is clear that Polygon 1 is similar to polygon 3 and 4 as they have the same shape but different sizes.

Therefore Polygon 3 and Polygon 4 are similar to polygon 1.

Learn more about similar polygon

here: https://brainly.com/question/1493409

#SPJ10

f(x)=x^2. which of these is g(x) ?

a. g(x)=(1/5x)^2
b. g(x)=5x^2
c.g(x)=(1/4x)^2
d.g(x)=1/5x^2

Answers

Answer:

a

Step-by-step explanation:

we know that g(x) means y therefore in this case let's suppose we have the coordinate (5;y) so we must find the corresponding y value

*DO THE TRIAL AND ERROR METHOD*

take any equation above and substitute the value of x of which is 5 and the corresponding value should be 1.

[tex]g(x) = (\frac{1}{5} x) {}^{2} [/tex]

An architect is designing new housing structures for the primate section at the zoo. Her plan is shown below. Which animals will live in a building that is similar to the main primate house? A orangutans B chimpanzees C gibbons D gorillas

Answers

Answer:

A

Step-by-step explanation:

:p

The expression 55 + 14m - 2n + 3p has _________ terms.

1. 3
2. 4
3. 2
4. 5

Answers

The expression has 3 terms

During a catered lunch, an average of 4 cups of tea are poured per minute. The lunch will last 2 hours. How many gallons of tea should the caterer bring if there are 16 cups in one gallon?

Answers

Answer:

30

Step-by-step explanation:

4 cups x 60 hours a min x 2 hours of lunch ÷ 16 cups per gallon = 30 gallons

The answer is 30 gallons.

Firstly, convert the hours to minutes so you know how many minutes you're working with. 2*60=120 minutes.

Then, multiply 120 with 4 to get the number of tea (in cups) that is poured. 120*4=480 cups.

Finally, divide 480 cups with 16 cups to find out the number of gallons poured. 480/16 equals 30 gallons.

Hope this helped.

What is a 10% margin increase on 1,810.69?

Answers

Answer:

1991.759

Step-by-step explanation:

1810.69×%10=181.069

1810.69+181.069=1991.759

1 in = 2.54 cm
how many millimeters are in 10.5 feet?
A.266.7 mm
B. 1,260 mm
C. 320.04 mm
D. 3,200.4 mm

Answers

Answer:

[tex]\fbox {D. 3,200.4 mm}[/tex]

Step-by-step explanation:

Given :

[ 1 inch = 2.54 centimeters ]

Unit conversions to keep in mind :

1 feet = 12 inches1 cm = 10 mm

Solving

10.5 feet10.5 x 12 inches126 inches126 x 2.54 cm320.04 cm320.04 x 10 mm3200.4 mm
The answer is D 3,200.4 mm I get that answer be equal ovulating the millimeters by 10.5 feet divided by the equal force of us in typical jetstream. Therefore 1 inches 2.54 cm. Then if I divide that by two, I get 3,200.4.

f(x) = x + 6 , find the ordered pair when x = -2.

(-2,4)

(2,-4)

(-2,-4)

(2,4)

Answers

-------------------------------------------------------------------------------------------------------------

Answer:  [tex]\textsf{Option A, (-2, 4)}[/tex]

-------------------------------------------------------------------------------------------------------------

Given:  [tex]\textsf{f(x) = x + 6}[/tex]

Find:  [tex]\textsf{f(-2)}[/tex]

Solution:  We need to plug in the value -2 into every variable x that we have in the expression and simplify to get the solution.

Plug in the values

[tex]\textsf{f(x) = x + 6}[/tex][tex]\textsf{f(-2) = -2 + 6}[/tex]

Simplify

[tex]\textsf{f(-2) = -2 + 6}[/tex][tex]\textsf{f(-2) = 4}[/tex]

Therefore, the ordered pair that would match would have a x-coordinate of -2 and a y-coordinate of 4 producing (-2, 4) which matches option A.

Give ur answer in standard form.
3 x 10^4 ÷6 x 10^-4

Answers

Answer:

5.0×10⁷

Step-by-step explanation:

(3÷6)×(10⁴÷10^-4)

(0.5)×(10⁸)

5.0×10^-1×10⁸

5.0×10⁷

[tex] \frac{3 \times {10}^{4} }{6 \times {10}^{ - 4} } \\ \\ \frac{1 \times {10}^{4 - ( - 4)} }{2 } \\ \\ \frac{ 1 \times {10}^{4 + 4} }{2} \\ \\ \frac{1 \times {10}^{8} }{2} \\ \\ 0.5 \times {10}^{8} .[/tex]

if x + 3y = 25 write y in terms of x and also find the two solutions of this equation

Answers

Answer:

y= 25/3 - x/3

Step-by-step explanation:

y= 25/3 - x/3

What is the sum of the arithmetic sequence 3, 9, 15..., if there are 24 terms? (5 points)

Answers

There's a fixed difference of 6 between terms (9 - 3 = 6, 15 - 9 = 6, and so on). The first term of the sequence is 3, so the [tex]n[/tex]-th term is

[tex]3 + 6(n-1) = 6n - 3[/tex]

If there are 24 terms in the sum, then the last term is 6×23 - 3 = 135.

Let [tex]S[/tex] be the sum,

[tex]S = 3 + 9 + 15 + \cdots + 123 + 129 + 135[/tex]

Reverse the order of terms:

[tex]S = 135 + 129 + 123 + \cdots + 15 + 9 + 3[/tex]

If we add up the terms in the same positions, we get twice [tex]S[/tex] on the left side, while on the right side we observe that each pair of terms will sum to 138.

[tex]S + S = (3 + 135) + (9 + 129) + (15 + 123) + \cdots + (135 + 3)[/tex]

[tex]2S = 138 + 138 + 138 + \cdots + 138[/tex]

and since there are 24 terms in the sum, the right side is the sum of 24 copies of 138. In other words,

[tex]2S = 24 \times 138[/tex]

and solving for [tex]S[/tex] gives

[tex]S = \dfrac{24\times138}2 = \boxed{1656}[/tex]

f(X)= 4X^2 + 7X -3 g(X) = 6X^3 - 7X^2-5 Find (f + g) (x).

Answers

By using the binary operator of addition, the result of summing f(x) = 4 · x² + 7 · x - 3 and g(x) = 6 · x³ - 7 · x² - 5 is equal to (f + g)(x) = 6 · x³ - 3 · x² + 7 · x - 8.

How to apply operations between functions

Binary operators is a operator that connects two functions. There are five binary operators between two functions: (i) Addition, (ii) Subtraction, (iii) Multiplication, (iv) Division, (v) Composition.

In this question we must apply the addition between two quadratic functions. In addition, we know by algebra that the sum of a quadratic function and a cubic function is equal to a cubic function. Hence, the resulting expression is:

(f + g)(x) = f(x) + g(x)

(f + g)(x) = (4 · x² + 7 · x - 3) + (6 · x³ - 7 · x² - 5)

(f + g)(x) = 6 · x³ - 3 · x² + 7 · x - 8

By using the binary operator of addition, the result of summing f(x) = 4 · x² + 7 · x - 3 and g(x) = 6 · x³ - 7 · x² - 5 is equal to (f + g)(x) = 6 · x³ - 3 · x² + 7 · x - 8.

To learn more on functions: https://brainly.com/question/21145944

#SPJ1

The correlation in error terms that arises when the error terms at successive points in time are related is termed _____.

Answers

Answer:

auto correlation is the answer to the question

The quadrilateral below is formed from a parallelogram and an
isosceles triangle.
Calculate the size of angle VQU.

Answers

Answer:

VQU =58°

Step-by-step explanation:

angle QUV is 61 because of F angles (I don't know what they call them in ur country)

triangle QUV is an isosceles triangles so base angles are the same

61+61=122

180-122=58

Add.

(3+x3+3x2)+(2x3−2−4x2)



Express the answer in standard form.

Enter your answer in the box.

Answers

Step-by-step explanation:

(3+3+3*2)+(2*3-2-4*2)

The value of the addition of the two polynomials is 3x³ - x² + 1.

What is addition of polynomial?

The addition of polynomials involves combining like terms of two or more polynomials to create a new polynomial. A polynomial is an expression consisting of variables, coefficients, and exponents, connected by addition and subtraction operations.

To add the given polynomials:

(3 + x³ + 3x²) + (2x³ - 2 - 4x²)

First, let's group the like terms:

(x³ + 2x³) + (3x² - 4x²) + (3 - 2)

Combine the coefficients of the like terms:

3x³ + 3x² - 4x² + 3 - 2

Simplify the expression:

3x³ - x² + 1

Therefore, the sum of the given polynomials is: 3x³ - x² + 1.

Learn more on addition of polynomial here;

https://brainly.com/question/10078818

#SPJ2

The vertex of this parabola is at (3-2). When the value is 4 the
y-value is 3. What is the coefficient of the squared expression in the
parabola's equation?
A. 7
B. 1
C. -1
D. 5

Answers

Answer:

D

Step-by-step explanation:

the equation of a parabola in vertex form is

y = a(x - h)² + k

where (h, k ) are the coordinates of the vertex and a is a multiplier

here (h, k ) = (3, - 2 ) , then

y = a(x - 3)² - 2

to find a substitute the point (4, 3 ) into the equation

3 = a(4 - 3)² - 2 ( add 2 to both sides )

5 = a × 1² = a

the coefficient of the squared expression is 5

Triangle AA'B'C' is the image of AABC under a dilation.

Answers

The scale factor is 3

Given: l || m; ∠1 ∠3

Prove: p || q

Horizontal and parallel lines l and m are intersected by parallel lines p and q. At the intersection of lines l and p, the uppercase left angle is angle 1. At the intersection of lines q and l, the bottom right angle is angle 2. At the intersection of lines q and m, the uppercase left angle is angle 3.

Complete the missing parts of the paragraph proof.



We know that angle 1 is congruent to angle 3 and that line l is parallel to line m because
. We see that is congruent to by the alternate interior angles theorem. Therefore, angle 1 is congruent to angle 2 by the transitive property. So, we can conclude that lines p and q are parallel by the

.

Answers

The parts that are missing in the proof are:

It is given

∠2 ≅ ∠3

converse alternate exterior angles theorem

What is the Converse of Alternate Exterior Angles Theorem?

The theorem states that, if two exterior alternate angles are congruent, then the lines cut by the transversal are parallel.

∠1 ≅ ∠3 and l║m because we are: given

By the transitive property,

∠2 and ∠3 are alternate interior angles, therefore, they are congruent to each other by the alternate interior angles theorem.

Based on the converse alternate exterior angles theorem, lines p and q are proven to be parallel.

Therefore, the missing parts pf the paragraph proof are:

It is given∠2 ≅ ∠3converse alternate exterior angles theorem

Learn more about the converse alternate exterior angles theorem on:

https://brainly.com/question/17883766

#SPJ1

Answer:

it is given

angle 2

angle 3

converse alternate exterior angles theorem

Step-by-step explanation:

PLEASE ANSWER ASAP!!!!!! WILL GIVE BRAINLIEST!!!

Answers

Answer ya so  1 is the answer

Step-by-step explanation:

Answer:

1

Step-by-step explanation:

Given expression:

[tex]\sf \left(\dfrac{3a^{-2}b^6}{2a^{-1}b^5} \right)^2[/tex]

To find the value of the expression when a = 3 and b = -2, substitute these values into the expression:

[tex]\implies \sf \left(\dfrac{3(3)^{-2}(-2)^6}{2(3)^{-1}(-2)^5} \right)^2[/tex]

[tex]\textsf{Apply exponent rule} \quad a^{-n}=\dfrac{1}{a^n}[/tex]

[tex]\sf \implies \left(\dfrac{3\left(\dfrac{1}{3^2}\right)(-2)^6}{2\left(\dfrac{1}{3^1} \right)(-2)^5} \right)^2[/tex]

[tex]\sf \implies \left(\dfrac{3\left(\dfrac{1}{9}\right)(-2)^6}{2\left(\dfrac{1}{3} \right)(-2)^5} \right)^2[/tex]

[tex]\sf \implies \left(\dfrac{\left(\dfrac{3}{9}\right)(-2)^6}{\left(\dfrac{2}{3} \right)(-2)^5} \right)^2[/tex]

[tex]\sf \implies \left(\dfrac{\left(\dfrac{1}{3}\right)(-2)^6}{\left(\dfrac{2}{3} \right)(-2)^5} \right)^2[/tex]

[tex]\textsf{Apply exponent rule} \quad (-a)^n=a^n,\:\: \textsf{ if }n \textsf{ is even}[/tex]

[tex]\textsf{Apply exponent rule} \quad (-a)^n=-a^n,\:\: \textsf{ if }n \textsf{ is odd}[/tex]

[tex]\sf \implies \left(\dfrac{\left(\dfrac{1}{3}\right) (2^6)}{\left(\dfrac{2}{3} \right) (-(2^5))} \right)^2[/tex]

[tex]\sf \implies \left(\dfrac{\left(\dfrac{1}{3}\right) (64)}{\left(\dfrac{2}{3} \right) (-32)} \right)^2[/tex]

[tex]\sf \implies \left(\dfrac{\left(\dfrac{1 \times 64}{3}\right)}{\left(\dfrac{2 \times -32}{3} \right)} \right)^2[/tex]

[tex]\sf \implies \left(\dfrac{\dfrac{64}{3}}{\dfrac{-64}{3}} \right)^2[/tex]

When dividing fractions, flip the second fraction and multiply it by the first:

[tex]\implies \sf \left( \dfrac{64}{3} \times \dfrac {3}{-64} \right)^2[/tex]

[tex]\implies \sf \left( \dfrac{64 \times 3}{3 \times (-64)}\right)^2[/tex]

[tex]\implies \sf \left( \dfrac{192}{-192}\right)^2[/tex]

[tex]\implies \sf \left(-1\right)^2[/tex]

[tex]\textsf{Apply exponent rule} \quad (-a)^n=a^n,\:\: \textsf{ if }n \textsf{ is even}[/tex]

[tex]\sf \implies 1^2=1[/tex]

Other Questions
a recursive formula for the sequence 18,9,4.5,... is In what three Central American countries was the Maya civilization most present? you and another provider have recognized that an adult patient is in cardiac arrest. an advanced airway is not in place. which actions demonstrate appropriate care? this stationary wave is what we call the first harmonic of the first normal mode of the system. in units of l, the length of the wire, what is 1, the wavelength of the first harmonic? I give you 12 points if you help me ;) its math I'm trying to find the name of a song from swing band music that encouraged audience participation & everyone stood up and said a few words with the band on the chorus? What is the mode of 1,1,2,2,3,3,4,4 The Russian region that slopes upward from the coast and has many streams, forests, and mountain ranges is known as the __________. Using a directrix of y = 2 and a focus of (2, 6), what quadratic function is created?here are the answers given \/ f(x) = one eighth (x 2)2 2 f(x) = one sixteenth (x 2)2 + 2 f(x) = one eighth (x 2)2 2 f(x) = one sixteenth (x + 2)2 2 A date for creation or revisionis mandatory for all web pages? How do smaller political scales (e.g., local governments or indigenous groups) influence human-environmental interactions and processes? Simplemente vaya a su pgina de preferencias y busque la opcin de idioma apropiado. In BCD, BP=15 cm.What the length of BX? 1. Henry is working in a pack room. He is filling a bag with dry seasoning. The net weight of the bag is stated as 35lbs. His tolerance is +/-5 lbs. Which of the following weights are allowable? Circle all allowable weights.a. 34b. 36C. 29d. 40 The Whigs were a political party that was part of the second party system.The party was created to challenge Jacksonian Democrats. Members included other democrats as well as the Anti-Masonic Party, who were against unfair treatment of Native Americans. Jackson had signed the _____ which he enforced strictly by forcing thousands of Native Americans to Indian Territory on foot, which was known as the ______ . what is the MOST common cause of airway obstruction in an unconscious patient? What's one of the most common reasons for a claim being rejected by an insurance company?Question 2 options:A) The patient's insurance information has been entered incorrectly.B) The patient is outside of the age group that is specified for the medication.C) The prescribing physician is out of the patient's network.D) The medication prescribed isn't the correct treatment for the patient's condition. Select the correct answer. which event most likely influenced the creation of the science fiction genre? a. the industrial revolution b. world war i c. the civil war d. the gold rush Identify the word choice that best reflects the eloi people in the time machine: Thatnight changed everything. It had become dangerousforme to live in Tylicz. . . . Papa deliberated longand hard over whomto contact about smugglingme across the border.Andrzej had beenfighting the Germans when Poland had first been invaded, but he'd been fortunateenough toescape capture; returning secretly to Tylicz, he was now workingfor the Polish resistance.Who knew the border better than Andrzej?..."I have a favor to ask of you, Andrzej .This iS very difficult for me, but I must ask. It is no longer safe inTylicz for Rena. Her mother and are worried for her safety every day."What assumptions can bemade about the setting described in the excerpt?O It takes place in present day Europe.O It takes place in Europe during the WWII era.O It takes place in present day United States.O It takes place in the United States in the WWIl era.Mark this and returnSave and ExitNextSubmit